Mathcenter Forum

Mathcenter Forum (https://www.mathcenter.net/forum/index.php)
-   อสมการ (https://www.mathcenter.net/forum/forumdisplay.php?f=18)
-   -   โจทย์ Inequality (https://www.mathcenter.net/forum/showthread.php?t=1462)

devilzoa 26 กุมภาพันธ์ 2007 20:48

โจทย์ Inequality
 
ผมอยากจะลองหาโจทย์ inequality มาลองฝึกทำดูน่ะครับ หาได้จากเว็บไหนหรือครับ

Mastermander 26 กุมภาพันธ์ 2007 21:13

ไม่ทราบว่าต้องการแบบไหนอะครับ

ลองดู Strategies มีอสมการนิดหน่อย

devilzoa 28 กุมภาพันธ์ 2007 12:55

แบบที่ให้พิสูจน์น่ะครับ...

Mastermander 28 กุมภาพันธ์ 2007 17:32

แปะให้สามข้อละกัน ไม่รู้ว่าเคยเห็นรึยัง

1. ให้ $a_1,a_2,\dots,a_n$ เป็นจำนวนจริงบวก ให้ $S=a_1+a_2+\dots+a_n$ โดยที่ $n>1$

จงแสดงว่า
$$(1+a_1)(1+a_2)\dots(1+a_n)<1+S+\dfrac{S^2}{2!}+\dots+\dfrac{S^n}{n!}$$

2. สำหรับจำนวนจริงบวก $a\,b\,c$ จงแสดงว่า $(a+b)(a+c) \ge 2\sqrt{abc(a+b+c)}$

3. กำหนดให้ $x,y$ เป็นจำนวนจริงบวกและ $x+y=1$ จงแสดงว่า $(1+\dfrac1x)(1+\dfrac1y)\ge 9$

nooonuii 01 มีนาคม 2007 09:56

ลองเข้าไปดูที่นี่ครับ
Inequality Marathon
มีโจทย์ให้ทำเยอะพอสมควร แต่หลังๆกระทู้ตกไปเพราะไม่มีคนเล่นครับ ว่างๆผมก็จะขุดขึ้นมาเล่นเป็นครั้งคราว :D

อีกอันคือที่นี่
Mathlink
ถ้าทำในนี้ได้หมดก็เป็นเซียนอสมการได้เลยครับ :great:

devilzoa 02 มีนาคม 2007 16:24

ยังไม่เคยเลยครับ

devilzoa 03 มีนาคม 2007 19:08

ทำได้แต่ข้อ 3 ครับ ใช้ Am-Gm ได้ $ \frac{1}{xy}\ge4 $
กระจายวงเล็บดูแล้วเอาไปแทนค่าก็ได้ มากกว่าหรือเท่ากับ 9 ครับ

nooonuii 04 มีนาคม 2007 00:10

โจทย์ของน้อง Mastermander ค่อนข้างจะโหดไปนิดนึงสำหรับผู้เริ่มต้นครับ อย่างข้อ 1 นี่เป็นข้อสอบ APMO 1989 มาก่อน ถ้าสนใจอยากทำจริงๆเดี๋ยวผมไปขุดกระทู้ inequality marathon ขึ้นมาให้อีกรอบครับ :D

Mastermander 04 มีนาคม 2007 15:58

Latex มันจะไม่ทำงานเมื่อมี UBB Code อยู่ข้างใน

มากกว่าหรือเท่ากับใช้โค้ด \ge

น้อยกว่าหรือเท่ากับ \le

Mastermander 05 มีนาคม 2007 10:37

http://www.vcharkarn.com/include/vca....php?Pid=84791

nooonuii 07 มีนาคม 2007 09:55

อ้างอิง:

ข้อความเดิมของคุณ Mastermander:
http://www.vcharkarn.com/include/vca....php?Pid=84791
โจทย์เยอะมากๆเลยครับ :yum: ตอนนี้ผมทะยอยทำไปเรื่อยๆ ใครสนใจอยากรู้ข้อไหนก็บอกมาละกันครับ

ขอใส่ข้อที่ทำได้ไว้ก่อนนะครับ แล้วจะเข้ามาเติมเรื่อยๆ

24 25 42 49 55
56 58 62 63 64
68 70 74 81 82
83 85 86 92 94
99

100 101 102 107 108
115 116 117 119 120
124 125 126 127 134
146 147 148 149 150
152 153 155

เพิ่มอีกหกข้อ :yum:

7 13 26 31 46 50

เติมรอบสอง :yum:

3 4 10 13 22 33 40 50 57

เติมรอบสาม

2 23 38 45 60 106 137 142

Comment : โจทย์ข้อ 2 พิมพ์ผิดครับ อสมการทางขวามือต้องเปลี่ยน 1 เป็น 2

dektep 08 มีนาคม 2007 13:11

ผมอยากรู้เฉลยข้อ 82,24,107,125,155,40
ข้อ 46. ผมคิดว่าโจทย์น่าจะผิดครับ

dektep 08 มีนาคม 2007 13:16

ช่วยส่งเฉลยมาที่ penguin_follower@hotmail.com

nooonuii 08 มีนาคม 2007 14:56

อ้างอิง:

ข้อความเดิมของคุณ dektep:
ผมอยากรู้เฉลยข้อ 82,24,107,125,155,40
ข้อ 46. ผมคิดว่าโจทย์น่าจะผิดครับ

ตอนนี้ผมเขียนโจทย์ใส่เศษกระดาษไว้ครับ ขอเอามาลงไว้ที่นี่ละกันครับ คนอื่นจะได้ศึกษาวิธีคิดไปด้วย หรือไม่ก็ช่วยจับผิดให้ผมด้วยครับ

46. Let $x=\sqrt{a},y=\sqrt{b},z=\sqrt{c}$. Then $x^4+y^4+z^4=1$.

The inequality is equivalent to

$$\frac{x^2}{y^4+1}+\frac{y^2}{z^4+1}+\frac{z^2}{x^4+1}\geq \frac{3}{4}\Big(x^3+y^3+z^3\Big)^2.$$
By Cauchy-Schwarz inequality we get
$$\begin{array}{rcl} \displaystyle{ \frac{x^2}{y^4+1}+\frac{y^2}{z^4+1}+\frac{z^2}{x^4+1} } & = & \displaystyle{ \frac{x^6}{x^4y^4+x^4}+\frac{y^6}{y^4z^4+y^4}+\frac{z^6}{z^4x^4+z^4} } \\ & \geq & \displaystyle{ \frac{(x^3+y^3+z^3)^2}{(x^4y^4+y^4z^4+z^4x^4)+(x^4+y^4+z^4)} } \\ & \geq & \displaystyle{ \frac{(x^3+y^3+z^3)^2}{\frac{1}{3}(x^4+y^4+z^4)^2+(x^4+y^4+z^4)} } \\ & = & \displaystyle{ \frac{3}{4} \Big(x^3+y^3+z^3\Big)^2. }\end{array}$$

dektep 08 มีนาคม 2007 15:13

ขอโทษครับสงสัยผมจะจำผิดข้อ :nooo:
ข้อที่โจทย์ผิดคือข้อ 21,5,32,79,140(มีคนบอกมา)
คุณ nooonuii ลองทำข้อ 1,97,161 ดูสิครับยากดี


เวลาที่แสดงทั้งหมด เป็นเวลาที่ประเทศไทย (GMT +7) ขณะนี้เป็นเวลา 13:05

Powered by vBulletin® Copyright ©2000 - 2024, Jelsoft Enterprises Ltd.
Modified by Jetsada Karnpracha